2018 AMC 12B Problems/Problem 6

Revision as of 17:09, 16 February 2018 by Giraffefun (talk | contribs) (Solution 1)

Problem

Suppose $S$ cans of soda can be purchased from a vending machine for $Q$ quarters. Which of the following expressions describes the number of cans of soda that can be purchased for $D$ dollars, where 1 dollar is worth 4 quarters?

$\textbf{(A)} \frac{4DQ}{S} \qquad \textbf{(B)} \frac{4DS}{Q} \qquad \textbf{(C)} \frac{4Q}{DS} \qquad \textbf{(D)} \frac{DQ}{4S} \qquad \textbf{(E)} \frac{DS}{4Q}$

Solution 1

The unit price for a can of soda (in quarters) is $\frac{S}{Q}$. Thus, the number of cans which can be bought for $D$ dollars ($4D$ quarters) is$\boxed {\textbf{(B)} \frac{4DS}{Q}}$ (Giraffefun)

See Also

2018 AMC 12B (ProblemsAnswer KeyResources)
Preceded by
Problem 5
Followed by
Problem 7
1 2 3 4 5 6 7 8 9 10 11 12 13 14 15 16 17 18 19 20 21 22 23 24 25
All AMC 12 Problems and Solutions

The problems on this page are copyrighted by the Mathematical Association of America's American Mathematics Competitions. AMC logo.png